-
-
Notifications
You must be signed in to change notification settings - Fork 78
New issue
Have a question about this project? Sign up for a free GitHub account to open an issue and contact its maintainers and the community.
By clicking “Sign up for GitHub”, you agree to our terms of service and privacy statement. We’ll occasionally send you account related emails.
Already on GitHub? Sign in to your account
combined uncertainty from multiple independent measurements? #38
Comments
The Does this answer your question? |
@lebigot yes and no :) is there any example I can follow for this particular combined uncertainty? |
Maybe I don't understand the question: it looks to me like everything is explained in the documentation, so I am not sure what the blocking point is. Let me have a stab at it, though: the question on Stack Overflow considers a few random variables: we can take for example
If you calculate their average,
You can check that the formula from the question gives the same result (after you transform the incorrect denominator 1/N into the correct 1/N^2)—which you don't have to calculate, thanks to
Does this clear things up? |
Yes, definitely! Now let me look at this closer myself. On Tue, Feb 2, 2016, 11:09 AM Eric O. LEBIGOT (EOL) <
|
PS: The formula for the uncertainty in the question is incorrect: the denominator should be N^2, not N. PPS: |
What happens when uncorrelated uncertainties are combined? |
Also your example does not seem correct, the combined value is not arithmetic average, the weights in the sum are based on corresponding uncertainties. |
As I was writing, I was translating for you the math form the question, not the math from the answer that you just referred to. If you want to translate the math from Steve B's answer, again, as I was writing in my first answer above, you simply replace the average If you want to know in general how uncorrelated uncertainties are combined by |
Sorry to wake up a 9 years old issue, but I was too wondering about this subject, and I too would have liked if a Would that be acceptable as a PR @lebigot? |
I have three points regarding this:
|
1/2. Yes I meant the maximum likelihood that is also mentioned here. You are correct that the formula I wrote assumes the correlations are all Here is the revised formulas for the uncertainty based on a complementary answer in the same SO Q&A, and with the same formula as before for the weighted mean: Where of course I think that implementing the above is very non-trivial, and it is worth adding it therefor to uncertainties. I have also managed to generalized it to averaging over any set of axes in #265 . |
Can this package handle uncertainty calculations like this?
http://physics.stackexchange.com/questions/57317/multiple-measurements-of-the-same-quantity-combining-uncertainties
If not, why?
The text was updated successfully, but these errors were encountered: